Transformation A Quadrilateral To A Rectangle.: Solution 1.

Download as docx, pdf, or txt
Download as docx, pdf, or txt
You are on page 1of 16

Transformation a quadrilateral to a rectangle.

Let

be a quadrilateral.

line parallel to

circle with diameter

perpendicular to
point on

be an arbitrary point on the

cuts the line through


.

Prove

that

cuts the line through

and parallel to
the

line

through

and
at a

carries the figure

into the figure

goes then to the intersection

to

parallel

figure
and

at O.

and perpendicular to

Solution 1.- Consider the homology fixing the pencil

line

is an arbitrary

from

As a result,
to

and

perpendicular to

to

is

and the line

It

The infinite point of the


of

therefore

with the parallel through


the

goes to the point

limiting

line

of

the

where the parallels from

intersect, in other words the intersection of the


at

and the perpendicular to

at

lies on

Comment: Can you explain to me some concepts: homology, infinite point,


limiting line? Thank you in advance
Answer: The concept of point at infinity is exactly the same thing as direction
in the plane, e.g. a set of parallel lines (having the same direction) is a set of
lines passing through a point at infinity. All points at infinity lie on the line at
infinity. The Euclidean plane with this line added is known as projective plane.
So here, any two

lines always

A bijection between two figures

intersect, parallels are no exception.


such that a point and an indicent line

go to a point a an indicent line, is called homography. The limiting line is


simply the image of the line at infinity under a homographic transformation.

Hence, in general there are 2 limiting lines


figure

in a bijection, one for each

A homology is a homography that fixes a line.

PQ is parallel to BC-2013 China TST Quiz 1 Day 1 P1

The quadrilateral
of
be

and
and

is inscribed in circle

and

meet at

, respectively. Let

parallel to

and

Now let

and
and

only meets segment

only meets segment

at

at

, and

, prove that

is

and

If

this

and
is

First we observe that


and

on

be the midpoints of

Solution 1.- Let the midpoints of sides


that

is the intersection point

. Let the projection of

respectively. If the circumcircle of


the circumcircle of

for

and

proved

we

I will prove
are

done.

lie on a circle centered at


the

same

be the reflection of

Observe

be

reason.

over

.
is a parallelogram.

that

and

ow we make the important observation that the two figures


and

are

homothetic

centering

and N to

mapping

to

to

to

. Hence
proving

that

are

concyclic .Done!
Comment 1: what do they mean by meet only at p and q? like its tangent? or
meet the segment
Answer to comment: meet at segment

Solution 2: Actually, it's very simple if you can see that


=

(Stewart's theorem),

triangle

and

||

||

is congruent to

and Q is the midpoint of

and P is the midpoint of

the

be the midpoints of
intersection

of
,

so

, which means

Solution 3.- Let


of

. Thus

Solution 2.- Let


of

, triangle

. Then

Similarly, on the other side


||

the

so

lies

, so

the circumcircle

with

on

as

Then
well.

be the midpoints of
intersection

, and

, and
of

with

the circumcircle
.

Then

,
so

, which means

so

lies

, so

on

as

well.

Euler line
Let ABC be a scalene triangle and
onto

the reflections of

..Let D be the intersection of

.Prove that that

is parallel to Euler line


Solution.- First we show O is on AD.
concyclic.
Now let

meet

at
c

oncyclic,

and

hence

Antigonal conjugate of I wrt its cevian triangle lies on O'I


Let

be

intersections of

triangle
,

antigonal conjugate of
where

and

let

be

its

with the sides

incenter.
,

, and

with respect to triangle

is the circumcenter of triangle

Let

as
be the projection of

.
on

be

the

. Prove that the


lies on the line

Solution 1.- Hope this will be helpful


Rewrite

Let

Let

be the projection of

Let

be the midpoint of

Invert

with

respect

to

on

.
.

and

denote

as

inversion.Easy to see
of

image

of

and

under

this

is the midpoint

Since

all lie on

the projection of

on

on

Since

harmonic quadrilateral. ie.


Similarly, we can get

,so

of

are collinear and

. Similarly, we can get

is the midpoint of

with respect to

Since

pass

the

is

.
get

is the reflection

through

the

midpoint

is the nine point circle of triangle

of triangle

so

is the midpoint of
we can

is

is the projection of
,

From the definition of

so

the

. ie.

. Since the Antigonal conjugate of


Poncelet

point

of

of

is the nine point center


is the reflection of
and

lie

in
on

,
so we get the image of the Antigonal conjugate of
and

is the midpoint of

is the common point of

Anti-steiner point of Euler line (called Kiepert focus which is


triangle

. (I use a property of Anti-steiner point here: If

which is the
in ETC ) of
is

the

reflection

of

with

respect

to

then

concur at the Anti-steiner point of the


line pass through
triangle
on

the

and the orthocenter of triangle

). Since
line

pass

is the pedal circle of

through

and

the

Kosnita

with respect to
, so the image of

point

of

lie

triangle

( Kosnita point is the isogonal conjugate of the nine point center which is
in ETC ). Now we only have the show

are collinear ... this

is a well known fact but I don't have the synthetic proof yet
Comment:

Now

we

only

have

to

prove

the

following

theorem

Theorem:
Let

be the nine point center of

Let

be the Kosnita point of

Let

be the Kiepert focus of

Then

are collinear

Proof:
Let

be the circumcenter of

Let

be the orthocenter of

Let

be the midpoint of

Let

be the reflection of

.
, respectively .

in

, respectively .

is the Anti-steiner point of the Euler line of

Feuerbach
of

be the tangential triangle of

Let

Since

point

of

Since

is

, so
the

is the

circumcenter

, respectively . So from the property of the Kosnita

point we get
and

. Consider the homothetic with center

factor

hence the image


of
Since

get

of

under this homothetic is

.
is the incenter of

(Antigonal
so

then

conjugate

of

, so the image
the

incenter).

Since

of
is

from http://www.artofproblemsolving.com/Foru
are collinear, hence

is
the
...

of
of

6&t=612271 we

are collinear .

Q.E.D
Problem of conic and three circles!
Let
Let

are concyclic;

are concyclic;

are on conic. Prove that:

are concyclic.
are concyclic.

Answer.given

Label
conic

axes

the
with

Arbitrary

circle w

through A,

D cuts C again at
are points on
such that the tangents
of
to

respectively.

and

By

Hence,
when

is either on

Let

generalized

or

varies, keeping

the

intersection
power

and
fixed, all lines

As a result, we deduce that


at

be

of

at
of

these

point,

tangents

we

are equal inclined to

have

Thus

go through a fixed direction.

and

then

If
are concyclic.

An inequality in a square
I research a pure geometric solution without any calculation
1. ABCD a square
2. E a point on the segment AD
3. F the foot of the perpendicular to CE through B
4. M the midpoint of EF

are parallel

cuts

again

5. G the point of intersection of the parallel to BC through M with the


perpendicular bissector of BF.
Prove : AC < 2.FG.
Sincerely, Jean-Louis
Answer.- Courtesy of Clarence Chew:
We want to show that: 2BC2=AC2<4FG2=4FX2+4XG2=BF2+4XG2 (let projection
of G to BF be X)
If we draw a line parallel to BC at X intersecting CF at Y, then Y is midpoint of
CF. XYMG is parallelogram, hence XG=XY=CE/2.
So

we

have

CF2<ED2.

to

show

Using

2BC2<BF2+CE2.
similarity

Using

of

Pythagorean
BFC

and

theorem,
CDE,

CF/ED=BC/CE=CD/CE. Now the result is obvious by hypotenuse longest side of


triangle.
Answer 2.1. C* the point of intersection of CE and BG.
2. FC = EC*3. according to Pythagoras theorem applied to FBC* and DAC, we
are done (a + 1/a >= 2 with a strictly positive)
Sincerely, Jean-Louis
an inequality in a square
I research a pure geometric solution without any calculation
1. ABCD a square
2. E a point on the segment AD
3. F the foot of the perpendicular to CE through B

4. M the midpoint of EF
5. G the point of intersection of the parallel to BC through M with the
perpendicular bisector of BF.
Prove : AC < 2.FG.
Sincerely
Jean-Louis
Sol 1.- Courtesy of Clarence Chew:
We want to show that: 2BC2=AC2<4FG2=4FX2+4XG2=BF2+4XG2 (let projection
of G to BF be X). If we draw a line parallel to BC at X intersecting CF at Y, then
Y is midpoint of CF. XYMG is parallelogram, hence XG=XY=CE/2. So we have to
show

2BC2<BF2+CE2.

Using

Pythagorean

theorem,

CF2<ED2. Using similarity of BFC and CDE, CF/ED=BC/CE=CD/CE. Now the


result is obvious by hypotenuse longest side of triangle.
Isogonal conjugate of I wrt. its cevian triangle lies on OI
Let

be a triangle and let

intersections of
Prove

(synthetically)

triangle
triangle

lies

on

the

the

isogonal

line

, and

conjugate

where

is

of
the

the

, respectively.
with

respect

to

circumcenter

of

Solution 1.- Rewrite


of

with the sides

that

on

be its incenter. Denote by

. Let

as

. Let
be the projection of

be the projection
on

. Invert with respect to

and denote

as the image of

under this

inversion.
Easy

to

see

and

of

the

midpoint

Since

all

so

are

Similarly,

we

lie

collinear

can

and

get

is

Since

is
the

the

projection

projection

of

of

on

on

on

so

is the midpoint of

of

so

is

is

Since

Since

the
the

triangle

is

nine

center
,

so

. Similarly, we can get


the

nine

point

point
center

of

all
the

center

ie. the isogonal conjugate of

circle

is the midpoint
of

of
lie

on

of

triangle

triangle

the

Euler

and

are

with respect to triangle

line

of

collinear.
lie on

Q.E.D
Euler-Poncelet points in cyclic quadrilateral
Given a cyclic quadrilateral
be

the

triangles
points of

isogonal

and an arbitrary point


conjugates

of

with

. Let
relation

that

, respectively. Prove that the Euler-Poncelet


are collinear.

Solution 1.- My solution:


Lemma:
Let

be four points . Let

WRT

be the orthopole of the line


.

Then

are collinear .Proof of the lemma:

Let

be the projection of

on

, respectively .

Let

be the infinity point with direction

Let

be the infinity point with direction

Let

be the infinity point with direction

Then
From Pappus theorem (for

and

) we get

are collinear .
Similarly, we can prove

are collinear , so we get

collinear .
Back to the main problem:
Let

be the center of

Let

be the Poncelet point of

Let

be the Poncelet point of

are

Let

be the Poncelet point of

Let

be the Poncelet point of

Since

is

the

orthopole
,

get

of

respectively.

with
so

from

relation
the

that

lemma

we

are collinear . Q.E.D

Two Triads of Circles


A.P. Hatzipolakis and P. Yiu, Triads of circles, preprint
In triangle ABC, let (Oa), (O1) be the circles tangent to circumcircle at vertex A
and

the

incircle

externally

at point A' and internally at point A", resp. Similarly B', C' and B",C"

Which

of

the

following

triangles

are

perspective

1.

ABC,

A'B'C'

2.

ABC,

A"B"C"

3.

A'B'C',

A"B"C"

APH

Reference: A.P. Hatzipolakis and P. Yiu, Triads of circles (preprint)

My

solution:

Lemma:

Let

be

the

radical

axis

of

and

be

point

on

Let

be

the

line

passing

through

and

tangent

to

at

Let

be

the

line

passing

through

and

tangent

to

at

Then

pass through the exsimilicenter or insimilicenter

of

(This is just a special case of Poncelet theorem, so I'll not post the proof here )

Back

to

Let

be

Let

be

Let

the

main

the

incenter

the

be

circumcenter

the

Let

be

the

radical
pole

of

Similarly,

axis

of

of

of

WRT

From radical center theorem ( for


get

problem:

respectively

and

) we

.
we

can

prove

and

so

we

get

ie.

are
and

Since

is

concurrent

at

the

pole

are

WRT

perspective

tangent

so from the lemma we get

of

to

respectively

pass through the insimilicenter of

Similarly, we can prove

pass through the insimilicenter of

so

are

ie.

we

get
and

Q.E.D

and

at

are

Similarly, we can prove


ie.

concurrent

perspective

are concurrent at
are

of

of

perspective

You might also like